Questions

Multiple choice questions are not just simple ways to check factual knowledge. They can also include images or can be used to integrate understanding across multiple areas. Questions are automatically included at the bottom of related articles as revision but can also be accessed directly from this page. Find out more about MCQs.

746 results found
Question

Question 3024

What kind of material were hip replacements originally made out of?

Question

Question 3023

Which complication is seen here?

Question

Question 3021

Which of the following is NOT a cause of diffuse bone marrow hypercellularity?

Question

Question 3020

Which of the following would be in keeping with a “myeloma defining” focal lesion on WB-MRI?

Question

Question 3016

What is the term used to describe the peripheral rim of relative T1 hyperintense signal seen in bone and soft tissue abscesses?

Question

Question 3015

In the presence of any destructive monoarthropathy, what diagnosis must be excluded?

Question

Question 3014

According to the Society of Skeletal Radiology White Paper on Nomenclature for Musculoskeletal Infection, what term should be used for any hyperintense marrow on fluid-sensitive images (regardless of T1 signal pattern) adjacent to an ulcer, abscess, or sinus tract?

Question

Question 3008

In skeletally immature individuals, the adductor longus originates from which anatomical feature, alongside the insertion of the rectus abdominis and the conjoint tendon of the inguinal ligament?

Question

Question 3006

Which two muscle groups are implicated in an injury indicated by the superior cleft sign on groin imaging?

Question

Question 3007

According to the Doha agreement on terminology and definitions in groin pain in athletes (2015), which of the following is NOT one of the defined clinical entities for groin pain?

Question

Question 3004

Using the Weber classification system, the fracture shown below is classified according to its relationship with what structure?

Question

Question 3005

A tear of which ligament is shown on the ankle radiograph below?

Question

Question 3003

Which of the following muscles attach to the greater trochanter?

Question

Question 2995

A 90 year old woman presents with left hip pain and refusal to bear weight after a fall. The radiograph below was obtained. What is the most appropriate next step in managing the patient?

Question

Question 2994

What is the most important feature of this subcapital femoral neck fracture with respect to treatment planning?

Question

Question 2993

What function does the transverse ligament of the hip serve?

Question

Question 2991

What is the most likely mechanism of injury for the fracture shown below?

Question

Question 2990

What part of the right proximal femur is fractured in the case shown below?
Question

Question 2974

Which oblique lumbar radiographic technique is depicted here and which pars interarticularis would it be assessing?

Question

Question 2982

Lumbar radiographs in a 65-year-old male with lower back pain reveal an indistinct margin of the superior endplate of L4. Which of the following is most likely to account for this?

Updating… Please wait.

 Unable to process the form. Check for errors and try again.

 Thank you for updating your details.